Forum: Analoge Elektronik und Schaltungstechnik Widerstand zwischen zwei Punkten


von Graue Maus (Gast)


Angehängte Dateien:

Lesenswert?

1. Also da sei ein unendlich ausgedehntes Widerstandsnetzwerk (siehe 
Bild).
Alle Widerstände haben 1k.
Wie groß ist er meßbare Widerstand zwischen den rot umrandeten Punkten?
Diese Aufgabe soll mal Teil des Einstellungstests bei Google gewesen 
sein.
Hätte ich also nicht bestanden...
Trotzdem würde mich schon mal interressieren wie man das rechnen kann!
Leider war Google bei der Suche nach einer Lösung für diese Aufgabe auch 
nicht hilfreich.
Habe versucht, das mit LTspice zu simulieren...

2. Als ähnliche Aufgabe, mit wahrscheinlich ähnlichen Lösungsansatz 
würde mir folgendes einfallen:
Wenn es anstelle des unendlichen Widerstandsnetzwerkes jetzt eine 
unendlich ausgedehnte Platine währe, Kupfer 35µm, wie groß währe der 
Widerstand zwischen zwei 1m entfernten Punkten?
Würde die Größe der Kontaktstellen eine Rolle spielen?

von NN (Gast)


Lesenswert?

Wahrscheinlich 0

von Fralla (Gast)


Lesenswert?

Ich musste das mal in Mathe rechnen, bei Folgen und Reihen, doch ich 
erinnere mich nicht mehr wie.

von Lothar M. (Firma: Titel) (lkmiller) (Moderator) Benutzerseite


Lesenswert?

NN schrieb:
> Wahrscheinlich 0
Wahrscheinlich nicht...
Bzw. nur, wenn bestimmte Widerstände 0 sind...

von Fralla (Gast)


Lesenswert?

Mit einer unendllichen ankettung von Spannungsteilern kann man es 
einfach rechnen mit Z=R+R//Z, dann kommt man auf 1.618*R. Vielcht mit 
ähnlichem  Ansatz.

von Mr Obvious (Gast)


Angehängte Dateien:

Lesenswert?


von U.R. Schmitt (Gast)


Lesenswert?

Mr Obvious schrieb:
> nerd_sniping.png

ROFLROFLROFL.

von Xeraniad X. (xeraniad)


Lesenswert?


von Xeraniad X. (xeraniad)


Lesenswert?

Es wurde nach der Diagonale über zwei benachbarte Quadrate gefragt; ich 
möchte hier jedoch nur mal die Diagonale über ein Quadrat betrachten und 
habe mit exakten Werten für endliche "Schachbretter" begonnen. Leider 
kann ich die Gesetzmässigkeit nicht erkennen (und erst recht nicht 
herleiten).
Die Diagonale des Quadrates in der Mitte des 3×3-R-Gitters hat z. B. den 
Widerstand Rdiag = 5÷7 ·R.
Die zweite Spalte enthält jeweils die Anzahl Widerstände R.
1
+-+---------+----------------+--------+
2
|n|2·n·(n+1)|           Rdiag| Rdiag÷R|
3
+-+---------+----------------+--------+
4
|1|        4|      1      ·R |1.000...|
5
|3|       24|     5÷7     ·R |0.714...|
6
|5|       60|   331÷495   ·R |0.668...|
7
|7|      112|235623÷360161·R |0.654...|
8
| |         |                |        |
9
|∞|         |     2÷π     ·R |0.636...|
10
+-+---------+----------------+--------+
Es ist zu erkennen, wie der Diagonalwiderstand des zentralen Quadrates 
mit wachsender Seitenlänge n gegen den Grenzwert Rdiag = 2÷π ·R strebt.

von Xeraniad X. (xeraniad)


Lesenswert?

Guten Tag
Jetzt (~ 6½ Jahre später) habe ich die Berechnung des Widerstandes über 
die Diagonale (innerhalb eines) Quadrates nachvollziehen können. #7
https://www.techniker-forum.de/thema/widerstandsnetzwerk.107858/
schönen Tag

: Bearbeitet durch User
von Hoho (Gast)


Lesenswert?

Weshalb misst denn keiner einfach mal nach?

von Achim H. (anymouse)


Lesenswert?

Hoho schrieb:
> Weshalb misst denn keiner einfach mal nach?

Die Praktiker sind noch alle beim Löten eines unendlich ausgedehnten 
Widerstandsnetz.

von Ralf (Gast)


Lesenswert?

Mittlerweile kann man tatsächlich ein unendlich ausgedehntes 
Widerstandsnetzwerk nachbauen, wenn man mit 3 Stellen nach dem Komma 
zufrieden ist.

10 x 10 Widerstände für die waagerechten Zeilen und genauso viele für 
die senkrechten Zeilen, also insgesamt 200 x 1k Ohm Widerstände so 
zusammenlöten wie in dem oberen Bild.

Danach die so entstandene Fläche zu einer Röhre zusammenrollen und 
verlöten. Das Messergebnis wird sich erst nach der vierten Kommastelle 
verändern (etwas kleiner Wert). Das kommt aber dem asymptotischen Wert 
sogar noch entgegen.

Die verwendeten Widerstände mit einer Toleranz von 1% sollten aus 
verschiedenen Produktions-Losen bzw. Lieferanten verwendet werden.

Der Vorteil dieser Konstruktion ist, dass ich schnell auch mal zwischen 
zwei ganz anderen Punkten messen kann und sofort, ein für die Praxis, 
ausreichend genauen Wert ermitteln kann.

von Aber, aber (Gast)


Lesenswert?

wir leben doch in einer dreidimsionalen Umgebung und nicht in einer 
Ebene.
Wie sieht denn das Egebnis in der Realität (3-Dim) aus?

von Ralf (Gast)


Lesenswert?

Aber, aber schrieb:
> Wie sieht denn das Egebnis in der Realität (3-Dim) aus?

oo = 2 / pi² x R = 0,203

von Xeraniad X. (xeraniad)


Lesenswert?

Nein, ich werde es nicht tun.
Nein, ich werde das 3fache Integral nicht auswerten.
Nein, ich werde es nicht tun.
Eher friert die Hölle zu.

von Ralf (Gast)


Lesenswert?

Xeraniad X. schrieb:
> Nein, ich werde das 3fache Integral nicht auswerten.

Werte doch bitte mal eben das 3fache Integral aus. Dann haben wir für 
diesen Thread sogar für unser 3D-Universum in allen 3 Ebenen den 
mathematischen Beweis.

Dann bist Du im gesamten mikrocontroller.net das Mathe-Ass.

von Xeraniad X. (xeraniad)


Lesenswert?

Ja, dann kann ich mich bei Gockel bewerben und hab dann gratis Frühstück 
und Sch!*?... -Salär.
schönen Abend

von Ralf (Gast)


Lesenswert?

Xeraniad X. schrieb:
> schönen Abend

Wir wünschen Dir auch einen schönen Abend (denk mal drüber nach).
Wir sprechen uns jedenfalls morgen in gewohnter Frische wieder und haben 
einen Nachweis für die 0,203k Ohm dabei!

Ich muss jetzt auch Schluss machen (Günther Jauch fängt jetzt an).

von Xeraniad X. (xeraniad)


Lesenswert?

Viel Spass!

: Bearbeitet durch User
von Xeraniad X. (xeraniad)


Angehängte Dateien:

Lesenswert?

Guten Tag
Zum dem in Posts zuvor erwähnten, unendlichen, kubischen 3D-Gitter wird 
hier 
https://physics.stackexchange.com/questions/167870/variation-of-infinite-grid-of-ideal-one-ohm-resistors-what-would-be-the-equival 
ein Papier von cserti 2000 zitiert.
Selbst bin ich beim Versuch, den Widerstand zwischen den Knoten 
beidseits einer Würfel-Diagonale zu berechnen, nur so weit wie im Bild 
anbei dargestellt gekommen. Dabei kann ich das mittlere Integral (über 
ß) wegen dem Parameter cos(alpha) nicht auswerten.
schönen Tag

: Bearbeitet durch User
von Ralf (Gast)


Lesenswert?

Für Diejenigen die es interessiert (Würfel): Rges = (5/6) x 1k = 833 
Ohm.

Bei dem Integral kann ich Dir leider nicht weiterhelfen. Aber schön, 
dass wir uns heute in gewohnter Frische wieder sprechen und dass Du Dich 
damit tatsächlich beschäftigt hast. Respekt!

von Yalu X. (yalu) (Moderator)


Angehängte Dateien:

Lesenswert?

Ich habe mal LTspice mit endlichen Widerstandswürfeln unterschiedlicher
Größe gefüttert und aus dem Simulationsergebnis den Gesamtwiderstand für
ein mittig liegendes Punktepaar bestimmt, das in x-, y- und z-Richtung
jeweils den Abstand 1 hat, also die Raumdiagonale eines Elementarwürfels
bildet.

Damit dieser Elementarwürfel mittig im Gesamtwürfel liegen kann, habe
ich nur ungeradzahlige Kantenlängen betrachtet: n = 1, 3, 5 ... 19.

Jeden dieser Würfel habe ich in zwei Varianten rechnen lassen:

1. Der normale Würfel mit 3·(n+1)²·n Einzelwiderständen, im Folgenden
   "offen" genannt. Dessen Gesamtwiderstand fällt streng monoton mit n,
   da das Hinzufügen weiterer Einzelwiderstände den Gesamtwiderstand nur
   verkleinern kann.

2. Der gemäß Ralfs Vorschlag vom 24.04. 15:50 zu einem vierdimensionalen
   Torus zusammengerollte Würfel mit 3·(n+1)³ Einzelwiderständen, im
   Folgenden "geschlossen" genannt. Dieser wächst für n≥3 streng
   monoton. Einen Beweis dafür habe ich zwar nicht, bin mir aber
   trotzdem sicher ;-)

Auf Grund der genannten Monotonieeigenschaften ist der Gesamtwiderstand
des offenen Würfels eine obere und der des geschlossen eine untere
Schranke für den Gesamtwiderstand des unendlichen Würfels. Wie man im
Diagramm erkennen kann, scheinen zudem beide gegen einen gemeinsamen
Grenzwert bei etwa 0,418 zu konvergieren.

Für n=20 liegen die beiden Werte (offen 0,418596, geschlossen 0,418182)
nur 0,1% auseinander, sie stellen also schon eine gute Näherung für den
unendlichen Würfel dar. Noch genauer ist der Mittelwert aus beiden. Er
beträgt 0,418389 und liegt damit nur 0,02% neben dem im Artikel von
Cserti genannten Wert von 0,418305.

Das Ganze stellt einen ausgesprochenen Härtetest für LTspice dar: Für
n=19 besteht der Würfel aus 22800 (offen) bzw. 24000 (geschlossen)
Einzelwiderständen. Entsprechend lang waren die Rechenzeiten, nämlich
1,5h (offen) bzw. 6h (geschlossen). Dass der offene Würfel trotz der
gleichen Anzahl von Knoten und der ähnlichen Anzahl von Widerständen um
den Faktor 4 schneller als der geschlossene berechnet wird, hängt wohl
damit zusammen, dass im offenen Fall das System durch eine Bandmatrix
beschrieben werden kann, für deren Berechnung es optimierte Verfahren
gibt.

LTspice macht offensichtlich von solchen Verfahren Gebrauch und hat sich
auch bzgl. Speicherverbrauch sehr gut geschlagen. Deswegen in Hoch auf
die Entwickler von Spice und LTspice!

: Bearbeitet durch Moderator
von Xeraniad X. (xeraniad)


Lesenswert?

Damit ist der Raum-Diagonal-Wert 0.418305 mehrfach bestätigt: wertvoll 
für Vergleiche mit anderen Methoden.
Hier ist der von Yalu X. zuvor erwähnte Cserti-Artikel: 
https://arxiv.org/abs/cond-mat/9909120
pdf S. 8, (1,1,1): 0.418305.

BTW, inzwischen versuchte ich, für die Auswertung des einige posts zuvor 
(25.04.2017 12:40) erwähnten 3fach -Integrals die {hierfür 
prädestinierte} Residuen-Methode anzuwenden, was für das innerste 
Integral funktioniert, aber beim mittleren Integral über ß komme ich 
wegen der Wurzeln im Nenner immer noch nicht weiter...

: Bearbeitet durch User
von Erich (Gast)


Lesenswert?


von Xeraniad X. (xeraniad)


Lesenswert?

Danke Erich. Der Wert für (1,1,1) 0.418305 ist in dem von Dir erwähnten 
Papier auf S. 60 ebenfalls zu finden. Auf S. 62 wird erwähnt, dass der 
exakte Ausdruck das (als Pozenzreihe darstellbare) vollständige 
https://de.wikipedia.org/wiki/Elliptisches_Integral K(...) (im Quadrat) 
enthält. Möglicherweise werde ich ein Programm schreiben, welches diesen 
Ausdruck numerisch ermittelt.

: Bearbeitet durch User
von Xeraniad X. (xeraniad)


Lesenswert?

1
#include <stdio.h>
2
#include <math.h>
3
4
    /* g++ hw.cpp -o hw ; ./hw   ; rm hw */ 
5
6
7
  double K (double k) 
8
  { /* complete elliptic integral K(k) = F(pi/2;k) first kind */
9
    double const pi   = atan2 (0.0e0, -1.0e0);
10
    double       fn   = 1.0e0;   /*   n!  */
11
    double       f2n  = 1.0e0;   /* (2n)! */
12
    double       p2n  = 1.0e0;   /* 2^(2*n) */
13
    double       k2n  = 1.0e0;   /* k^(2*n) */
14
    double       _n   = 0.0e0;   
15
    double       _2n  = 0.0e0;   /* 2*n */
16
    double       res  = 0.0e0;
17
    for (int n = 0;   16 > n;  ++ n) {  /* converges fast */
18
      double const quo = f2n / (p2n * fn * fn);
19
      res += quo * quo * k2n;
20
      _2n += 1.0e0;
21
      f2n *= _2n;
22
      _2n += 1.0e0;
23
      f2n *= _2n;
24
      k2n *= k * k;
25
      p2n *= 4.0e0;
26
      _n  += 1.0e0;
27
      fn  *= _n;
28
   /* (void) fprintf (stdout, "n: %02d, res:  %0.14lf\n", n, res);  */
29
    }  /* n */
30
31
    res *= 0.5e0 * pi;
32
    return res;
33
  }  /* K */
34
35
  int main (int argc, char const * argv [])
36
  {
37
    double const pi = atan2 (0.0e0, -1.0e0);
38
    double const p1 =  9.0e0/8.0e0; 
39
    double const p2 = -3.0e0/4.0e0; 
40
    double const p3 =  0.0e0;
41
    double const k0 = (2.0e0-sqrt(3.0e0)) * (sqrt(3.0e0)-sqrt(2.0e0)); 
42
    double const K_k0 = K (k0);
43
    double const g0 = 4.0e0/(pi*pi) * (18.0e0 +12.0e0*sqrt(2.0e0) -10.*sqrt(3.0e0) -7.0e0*sqrt(6.0e0)) * K_k0*K_k0;
44
    double const r111 = p1 * g0 + p2 /(pi*pi*g0) + p3;
45
 /* (void) fprintf (stdout, "pi:    %0.14lf\n", pi);   */
46
 /* (void) fprintf (stdout, "k0:    %0.14lf\n", k0);   */
47
 /* (void) fprintf (stdout, "K_k0:  %0.14lf\n", K_k0); */
48
    (void) fprintf (stdout, "g0:    %0.14lf\n", g0); 
49
    (void) fprintf (stdout, "r111:  %0.14lf\n", r111); 
50
51
    return 0;
52
  }  /* main */
output:
1
g0:    0.50546201971733  
2
r111:  0.41830531092188
Die Werte entsprechen denen in von Erich zitiertem script S. 61 bzw. 60.

: Bearbeitet durch User
Bitte melde dich an um einen Beitrag zu schreiben. Anmeldung ist kostenlos und dauert nur eine Minute.
Bestehender Account
Schon ein Account bei Google/GoogleMail? Keine Anmeldung erforderlich!
Mit Google-Account einloggen
Noch kein Account? Hier anmelden.